RegistrierenRegistrieren   LoginLogin   FAQFAQ    SuchenSuchen   
Problem mit einer Schaltung ( Widerstände )
 
Neue Frage »
Antworten »
    Foren-Übersicht -> Elektrik
Autor Nachricht
Gimel



Anmeldungsdatum: 06.11.2007
Beiträge: 96
Wohnort: Wuppertal

Beitrag Gimel Verfasst am: 20. Mai 2008 15:02    Titel: Problem mit einer Schaltung ( Widerstände ) Antworten mit Zitat

Hallo Physiker!
Ich hab hier eine Schaltung vorliegen, deren Gesamtwiderstand ich berechnen soll. Ich weiß wie das bei Parallelschaltungen und Reihenschaltungen geht. Allerdings ist der vorliegende etwas komplex und ich bin mir nicht sicher, ob es da weitere Formeln gibt, die ich brauche.
Hier ist ein Bild von mir ( die Schaltung war noch komplizierter, ich hab sie - hoffentlich richtig Big Laugh - auf diesen Fall vereinfacht ) :



Schaltung.JPG
 Beschreibung:

Download
 Dateiname:  Schaltung.JPG
 Dateigröße:  18.86 KB
 Heruntergeladen:  1321 mal

dermarkus
Administrator


Anmeldungsdatum: 12.01.2006
Beiträge: 14788

Beitrag dermarkus Verfasst am: 20. Mai 2008 15:08    Titel: Antworten mit Zitat

Da verstehe ich nicht, wie du diese Schaltung meinst. Ein Widerstand ist doch etwas mit zwei Drähten dran, und nicht mit drei.

Magst du vielleicht doch lieber mal die ursprüngliche, noch nicht von dir vereinfachte Schaltung zeigen?
Gimel



Anmeldungsdatum: 06.11.2007
Beiträge: 96
Wohnort: Wuppertal

Beitrag Gimel Verfasst am: 20. Mai 2008 15:19    Titel: Antworten mit Zitat

Ein "Sagen" reicht da völlig aus. Augenzwinkern
Es handelt sich um einen Würfel und in jede seiner Kanten hat einen Innenwiderstand von 10 Ohm.
Dabei soll ich dann den Gesamtwiderstand zwischen zwei gegenüberliegenden Ecken berechnen.

Das mit den drei Drähten hat mich auch stutzig gemacht. Aber jetzt im Nachhinein glaube ich, dass ich es falsch gemacht hab, da mit Kanten nicht Ecken gemeint sind.... ich versuchs nochmal smile
Gimel



Anmeldungsdatum: 06.11.2007
Beiträge: 96
Wohnort: Wuppertal

Beitrag Gimel Verfasst am: 20. Mai 2008 15:48    Titel: Antworten mit Zitat

Ok, hab jetzt was neues.
Das Problem ist nur, dass ich die Widerstände nun mit "Drähten" verbunden habe. Aber laut aufgaben Stellung würden sich ja alle Widerstände berühren, da sie nunmal die Kanten sind.

Außerdem frage ich mich, ob es einen Unterschied macht, wie ich den Anschluss darstelle, daher zwei Bilder ( beim zweiten soll der Abstand andeuten, dass der eine Draht unter dem anderen verläuft Augenzwinkern )



Schaltung 1.JPG
 Beschreibung:

Download
 Dateiname:  Schaltung 1.JPG
 Dateigröße:  18.61 KB
 Heruntergeladen:  1252 mal


Schaltung 2.JPG
 Beschreibung:

Download
 Dateiname:  Schaltung 2.JPG
 Dateigröße:  19.63 KB
 Heruntergeladen:  1345 mal

dermarkus
Administrator


Anmeldungsdatum: 12.01.2006
Beiträge: 14788

Beitrag dermarkus Verfasst am: 20. Mai 2008 16:01    Titel: Antworten mit Zitat

Schaltung 1 macht noch keinen Sinn, aber Schaltung 2 scheint mir okay zu sein.

Aber ich würde eher vorschlagen, du zeichnest dir den Widerstandswürfel lieber dreidimensional auf, anstatt ihn ins Zweidimensionale zu quetschen.

Denn im Dreidimensionalen sieht man viel einfacher die Symmetrieeigenschaften, mit Hilfe deren man diese Schaltung vereinfachen kann. smile
Gimel



Anmeldungsdatum: 06.11.2007
Beiträge: 96
Wohnort: Wuppertal

Beitrag Gimel Verfasst am: 20. Mai 2008 16:08    Titel: Antworten mit Zitat

Oops! Bei Schaltung 1 bin ich auch ein opfer von Strg-C und Strg-V geworden. Da sind ja wieder 3 Drähte an einem Widerstand, hab das hier mal korrigiert.
Und auch gleich nochmal die Frage, ob dann jetzt ein Unterschied zwischen Schaltung eins und 2 vom Gesamtwiderstand her besteht?
( Wenn ich mir den Würfel aufzeichne, dann erkenne ich im dreidimensionalen nicht mehr wirklich, ob es parallel oder in Reihe geschaltet ist grübelnd ).



Schaltung 1.JPG
 Beschreibung:

Download
 Dateiname:  Schaltung 1.JPG
 Dateigröße:  19.94 KB
 Heruntergeladen:  1270 mal

dermarkus
Administrator


Anmeldungsdatum: 12.01.2006
Beiträge: 14788

Beitrag dermarkus Verfasst am: 20. Mai 2008 16:13    Titel: Antworten mit Zitat

Nein, diese Schaltung 1 kann nicht stimmen, denn da haben die Widerstände ganz links und ganz rechts ja nicht dieselben Verbindungen wie im dreidimensionalen Würfel.

Welcher Widerstand zu welchem nun parallel oder in Reihe ist, das sihst du ja aber auch in deiner zweidimensionalen Schaltung 2 sicher kaum.

Tipp zum Vereinfachen des dreidimensionalen Würfels: Überlege dir mal, welche Punkte aus Symmetriegründen jeweils alle auf gleichem Potential liegen müssen. Kannst du da irgendwelche Teile mit Hilfe von Spiegelsymmetrieüberlegungen zusammenlegen?
Gimel



Anmeldungsdatum: 06.11.2007
Beiträge: 96
Wohnort: Wuppertal

Beitrag Gimel Verfasst am: 20. Mai 2008 16:51    Titel: Antworten mit Zitat

So unter Betrachtung deines Tips, habe ich meine ( hoffentlich ) endgültige Schaltung entworfen:

Somit würde ich auf 20/3 Ohm kommen.



endgültige Schaltung.JPG
 Beschreibung:
Gleiche Zahlen = Gleiches Potential

In der Reihe mit Potential 3: Gleiche Farbe = Gleiche Kante

Download
 Dateiname:  endgültige Schaltung.JPG
 Dateigröße:  23.73 KB
 Heruntergeladen:  1272 mal

dermarkus
Administrator


Anmeldungsdatum: 12.01.2006
Beiträge: 14788

Beitrag dermarkus Verfasst am: 20. Mai 2008 16:56    Titel: Antworten mit Zitat

Mein Tipp: Zeichne dir das unbedingt perspektivisch dreidimensional auf.

Anhand einer zweidimensionalen Skizze würde ich die Symmetrien in diesem Würfel nicht oder nur viel, viel schwerer erkennen, und obendrein läuft man beim Aufzeichnen von solchen zweidimensionalen Versionen viel zu leicht Gefahr, Flüchtigkeitsfehler zu machen.

(Beispiel für so einen Flüchtigkeitsfehler: Du hast da 15 Widerstände eingezeichnet, aber der Würfel hat nur 12 Kanten.)
Neue Frage »
Antworten »
    Foren-Übersicht -> Elektrik